Bạn chưa đăng nhập. Vui lòng đăng nhập để hỏi bài

Những câu hỏi liên quan
Phạm Đức Anh
Xem chi tiết
Kudo Shinichi
30 tháng 3 2016 lúc 6:15

a/2 >hoặc = a/5 ( xảy ra giấu bằng với a=0)

b/3> hoặc = b/5 ( xảy randaaus bằng với a=0

Do đó : a/2 +b/3 = a/5 + b/5 chỉ trong trường hợp a=b=0

Vũ Thị Hằng Nga
12 tháng 2 2017 lúc 16:22

tìm các số tự nhiên a,b,c sao cho a^2 <=b;b^2<=c;c^2<=a

Nguyễn Văn Khôi
28 tháng 10 2017 lúc 21:04

very easy

Đảo Rồng
Xem chi tiết
Đỗ Đường Hùng
23 tháng 1 2017 lúc 21:08

a=4,b=3

m=3,n=2

Nguyễn Thị Thúy Hường
Xem chi tiết
Lê Trung Kiên
19 tháng 2 2017 lúc 21:01

đáp án là:1/2+15+1/10=4/5

NGUUYỄN NGỌC MINH
Xem chi tiết
Hồ Lê Phú Lộc
15 tháng 7 2015 lúc 19:52

1+6/32+42=7/25

Hoàng Thị Minh Phương
Xem chi tiết
Trà My
Xem chi tiết
Giang Trường
3 tháng 3 2016 lúc 11:21

  từ giả thiết=> 2/b=a/5-2/15=(3a-2)/15 
=>b/2=15/(3a-2) (nghịch đảo hai vế) 
=>b=30/(3a-2) 
để b là số tự nhiên thì: 
a=1 =>b=30 => tích ab=30 
a=4 =>b=3 => tích ab=12 
KL: tích ab lớn nhất =30

Vũ Tường Minh
Xem chi tiết
Hoàng Thị Thanh Huyền
31 tháng 3 2018 lúc 12:07

\(\frac{52}{9}=5+\frac{1}{a+\frac{1}{b+\frac{1}{c}}}\)

\(\frac{52}{9}=5+\frac{7}{9}=5+\frac{1}{\frac{9}{7}}\)

\(=5+\frac{1}{1+\frac{2}{7}}\)

\(=5+\frac{1}{1+\frac{1}{\frac{7}{2}}}\)

\(=5+\frac{1}{1+\frac{1}{3+\frac{1}{2}}}\)

\(\Rightarrow a=1,b=3,c=2\)

nguyen to oanh
29 tháng 4 2018 lúc 12:53

huyen lam chuan day

kamehameha100
Xem chi tiết
Zlatan Ibrahimovic
2 tháng 4 2017 lúc 7:28

Ta có :4/5=8/10=(1+2+5)/10=1/10+2/10+5/10=1/10+1/5+1/2.

Vì a,b,c có vai trò như nhau =>a=10;b=5;c=2

kamehameha100
2 tháng 4 2017 lúc 7:23

ai làm đúng nhanh chi tiết thì mình k cho hehe

Ta có :4/5=8/10=(1+2+5)/10=1/10+2/10+5/10=1/10+1/5+1/2.

Vì a,b,c có vai trò như nhau =>a=10;b=5;c=2 

Ai đó k mik đi!

Trần Minh Hưng
Xem chi tiết
Nguyễn Huyền Minh
25 tháng 3 2017 lúc 20:46

VD tổng nghịch đâỏ cảu ba số này là 2 thì:
Số lớn nhất là a, số nhỏ nhất là c.
Ta có: c ≤ b ≤ a (1)
Theo giả thiết : \(\dfrac{1}{a}+\dfrac{1}{b}+\dfrac{1}{c}\) = 2 (2)
Do (1) nên 2 = \(\dfrac{1}{a}+\dfrac{1}{b}+\dfrac{1}{c}\)\(\dfrac{3}{c}\)
Vậy c = 1
Thay vào (2) ta dc :\(\dfrac{1}{a}+\dfrac{1}{b}\) = 1 ≤ \(\dfrac{2}{b}\)
Vậy a = 2 từ đó b = 2
3 số cần tìm là 1; 2; 2.